Why is the Ratio of ln(x) and log(x) a constant?












2












$begingroup$


I was solving some "Big-Oh" algorithm asymptotic complexity problems, when I discovered that for some constant $c$ and some variable $x$:



$$c^{log(x)}$$ and $$x^{log(c)}$$



grow at the same rate. When figuring this out I ended up with the expression:



$$frac{ln(x)}{log(x)}=frac{ln(c)}{log(c)}approx 2.3025$$



This result was surpirsing and somewhat baffling to me. This might seem like a naive question, but could someone help me understand how the ratio of $log_{10}(x)$ and $ln(x)$ ends up being a constant value?










share|cite|improve this question









$endgroup$












  • $begingroup$
    Indeed the expressions you gave are equal, not just of equal growth: $$c^{log(x)}=e^{log(c)log(x)}=x^{log(c )}$$
    $endgroup$
    – adfriedman
    2 hours ago


















2












$begingroup$


I was solving some "Big-Oh" algorithm asymptotic complexity problems, when I discovered that for some constant $c$ and some variable $x$:



$$c^{log(x)}$$ and $$x^{log(c)}$$



grow at the same rate. When figuring this out I ended up with the expression:



$$frac{ln(x)}{log(x)}=frac{ln(c)}{log(c)}approx 2.3025$$



This result was surpirsing and somewhat baffling to me. This might seem like a naive question, but could someone help me understand how the ratio of $log_{10}(x)$ and $ln(x)$ ends up being a constant value?










share|cite|improve this question









$endgroup$












  • $begingroup$
    Indeed the expressions you gave are equal, not just of equal growth: $$c^{log(x)}=e^{log(c)log(x)}=x^{log(c )}$$
    $endgroup$
    – adfriedman
    2 hours ago
















2












2








2





$begingroup$


I was solving some "Big-Oh" algorithm asymptotic complexity problems, when I discovered that for some constant $c$ and some variable $x$:



$$c^{log(x)}$$ and $$x^{log(c)}$$



grow at the same rate. When figuring this out I ended up with the expression:



$$frac{ln(x)}{log(x)}=frac{ln(c)}{log(c)}approx 2.3025$$



This result was surpirsing and somewhat baffling to me. This might seem like a naive question, but could someone help me understand how the ratio of $log_{10}(x)$ and $ln(x)$ ends up being a constant value?










share|cite|improve this question









$endgroup$




I was solving some "Big-Oh" algorithm asymptotic complexity problems, when I discovered that for some constant $c$ and some variable $x$:



$$c^{log(x)}$$ and $$x^{log(c)}$$



grow at the same rate. When figuring this out I ended up with the expression:



$$frac{ln(x)}{log(x)}=frac{ln(c)}{log(c)}approx 2.3025$$



This result was surpirsing and somewhat baffling to me. This might seem like a naive question, but could someone help me understand how the ratio of $log_{10}(x)$ and $ln(x)$ ends up being a constant value?







logarithms






share|cite|improve this question













share|cite|improve this question











share|cite|improve this question




share|cite|improve this question










asked 3 hours ago









user3776749user3776749

286210




286210












  • $begingroup$
    Indeed the expressions you gave are equal, not just of equal growth: $$c^{log(x)}=e^{log(c)log(x)}=x^{log(c )}$$
    $endgroup$
    – adfriedman
    2 hours ago




















  • $begingroup$
    Indeed the expressions you gave are equal, not just of equal growth: $$c^{log(x)}=e^{log(c)log(x)}=x^{log(c )}$$
    $endgroup$
    – adfriedman
    2 hours ago


















$begingroup$
Indeed the expressions you gave are equal, not just of equal growth: $$c^{log(x)}=e^{log(c)log(x)}=x^{log(c )}$$
$endgroup$
– adfriedman
2 hours ago






$begingroup$
Indeed the expressions you gave are equal, not just of equal growth: $$c^{log(x)}=e^{log(c)log(x)}=x^{log(c )}$$
$endgroup$
– adfriedman
2 hours ago












5 Answers
5






active

oldest

votes


















2












$begingroup$

For all $xneq1$, $x>0$ we have: $$frac{ln{x}}{log{x}}=frac{ln{x}}{frac{log_ex}{log_e{10}}}=log_e10=ln10.$$






share|cite|improve this answer











$endgroup$





















    1












    $begingroup$

    If $ln x = a_x$ and $log_{10} x = b_x$ then



    $e^{a_x} = x$ and $10^{b_x} = x$.



    Bear in mind $10 = e^{ln 10}$ so $10^k = (e^{ln 10})^k = e^{kln 10}$.



    So if $10^{k_x} =e^{k_xln 10} = x$ then......



    By definition $log_{10} x = k_x$ and $ln x = k_xln 10$ and so.......



    $frac {ln x}{log_{10} x} = frac {k_xln 10}{k_x} = ln 10$.



    It's just a conversion constant and shouldn't surprise us.



    This is the very basis of the rule $log_b x = frac {log_a x}{log_a b}$ (note if $x$ is a variable and $b$ is a constant that is exactly your observation).






    share|cite|improve this answer









    $endgroup$





















      0












      $begingroup$

      Hint: $ln(10)approx 2.3025$. Given that information, your conjecture is that $ln(x)=ln(10)log_{10}(x)$. Can you see a way to prove that?






      share|cite|improve this answer









      $endgroup$





















        0












        $begingroup$

        $log_a b$ = $log_c b over log_c a$ is a general rule.
        Thus $log_c a = {log_c b over log_a b}$ and your expression is the case of this when $a=10, b=x, c=e$.






        share|cite|improve this answer











        $endgroup$













        • $begingroup$
          Ummm.... by your correct rule we have $$ln(10)=frac{log_x(10)}{log_x(e)}$$ which isn't what we're trying to prove.
          $endgroup$
          – Rhys Hughes
          2 hours ago












        • $begingroup$
          Thanks. I edited to correct
          $endgroup$
          – J. W. Tanner
          2 hours ago



















        0












        $begingroup$

        Some of the answers already provided get close to a full explanation, but not quite.



        Recall that if $b > 1$ and $x > 0$, and $$log_b x = y,$$ what this means is that $b^y = x$. In other words, the base-$b$ logarithm of $x$ is an exponent $y$ such that when the base $b$ is raised to the $y^{rm th}$ power, the result is $x$. This is the definition of a (real-valued) logarithm.



        So, why is it that for two bases $a$, $b$, $$frac{log_a x}{log_b x}$$ is a constant not dependent on $x$? The reason is that $log_a x$ is an exponent, say $y$, such that $a^y = x$; and $log_b x$ is an exponent, say $w$, such that $b^w = x$; then $$b^w = x = a^y.$$ And now, raising both sides to the $1/w$ power, we get $$b = (b^w)^{1/w} = (a^y)^{1/w} = a^{y/w}.$$ So the ratio $y/w$ does not depend on $x$. In fact, again using the definition of logarithm, $y/w$ is the exponent for which the base $a$ must be raised to yield $b$; that is, we explicitly have $$frac{y}{w} = log_a b,$$ and from this, we get (with one additional algebraic step) what is known as the "change-of-base" formula $$frac{log_a x}{log_a b} = log_b x.$$



        Note that only the definition of $log$ was used, and the rule for exponents $(b^m)^n = b^{mn}$.






        share|cite|improve this answer









        $endgroup$













          Your Answer





          StackExchange.ifUsing("editor", function () {
          return StackExchange.using("mathjaxEditing", function () {
          StackExchange.MarkdownEditor.creationCallbacks.add(function (editor, postfix) {
          StackExchange.mathjaxEditing.prepareWmdForMathJax(editor, postfix, [["$", "$"], ["\\(","\\)"]]);
          });
          });
          }, "mathjax-editing");

          StackExchange.ready(function() {
          var channelOptions = {
          tags: "".split(" "),
          id: "69"
          };
          initTagRenderer("".split(" "), "".split(" "), channelOptions);

          StackExchange.using("externalEditor", function() {
          // Have to fire editor after snippets, if snippets enabled
          if (StackExchange.settings.snippets.snippetsEnabled) {
          StackExchange.using("snippets", function() {
          createEditor();
          });
          }
          else {
          createEditor();
          }
          });

          function createEditor() {
          StackExchange.prepareEditor({
          heartbeatType: 'answer',
          autoActivateHeartbeat: false,
          convertImagesToLinks: true,
          noModals: true,
          showLowRepImageUploadWarning: true,
          reputationToPostImages: 10,
          bindNavPrevention: true,
          postfix: "",
          imageUploader: {
          brandingHtml: "Powered by u003ca class="icon-imgur-white" href="https://imgur.com/"u003eu003c/au003e",
          contentPolicyHtml: "User contributions licensed under u003ca href="https://creativecommons.org/licenses/by-sa/3.0/"u003ecc by-sa 3.0 with attribution requiredu003c/au003e u003ca href="https://stackoverflow.com/legal/content-policy"u003e(content policy)u003c/au003e",
          allowUrls: true
          },
          noCode: true, onDemand: true,
          discardSelector: ".discard-answer"
          ,immediatelyShowMarkdownHelp:true
          });


          }
          });














          draft saved

          draft discarded


















          StackExchange.ready(
          function () {
          StackExchange.openid.initPostLogin('.new-post-login', 'https%3a%2f%2fmath.stackexchange.com%2fquestions%2f3089210%2fwhy-is-the-ratio-of-lnx-and-logx-a-constant%23new-answer', 'question_page');
          }
          );

          Post as a guest















          Required, but never shown

























          5 Answers
          5






          active

          oldest

          votes








          5 Answers
          5






          active

          oldest

          votes









          active

          oldest

          votes






          active

          oldest

          votes









          2












          $begingroup$

          For all $xneq1$, $x>0$ we have: $$frac{ln{x}}{log{x}}=frac{ln{x}}{frac{log_ex}{log_e{10}}}=log_e10=ln10.$$






          share|cite|improve this answer











          $endgroup$


















            2












            $begingroup$

            For all $xneq1$, $x>0$ we have: $$frac{ln{x}}{log{x}}=frac{ln{x}}{frac{log_ex}{log_e{10}}}=log_e10=ln10.$$






            share|cite|improve this answer











            $endgroup$
















              2












              2








              2





              $begingroup$

              For all $xneq1$, $x>0$ we have: $$frac{ln{x}}{log{x}}=frac{ln{x}}{frac{log_ex}{log_e{10}}}=log_e10=ln10.$$






              share|cite|improve this answer











              $endgroup$



              For all $xneq1$, $x>0$ we have: $$frac{ln{x}}{log{x}}=frac{ln{x}}{frac{log_ex}{log_e{10}}}=log_e10=ln10.$$







              share|cite|improve this answer














              share|cite|improve this answer



              share|cite|improve this answer








              edited 1 hour ago

























              answered 2 hours ago









              Michael RozenbergMichael Rozenberg

              99.5k1590189




              99.5k1590189























                  1












                  $begingroup$

                  If $ln x = a_x$ and $log_{10} x = b_x$ then



                  $e^{a_x} = x$ and $10^{b_x} = x$.



                  Bear in mind $10 = e^{ln 10}$ so $10^k = (e^{ln 10})^k = e^{kln 10}$.



                  So if $10^{k_x} =e^{k_xln 10} = x$ then......



                  By definition $log_{10} x = k_x$ and $ln x = k_xln 10$ and so.......



                  $frac {ln x}{log_{10} x} = frac {k_xln 10}{k_x} = ln 10$.



                  It's just a conversion constant and shouldn't surprise us.



                  This is the very basis of the rule $log_b x = frac {log_a x}{log_a b}$ (note if $x$ is a variable and $b$ is a constant that is exactly your observation).






                  share|cite|improve this answer









                  $endgroup$


















                    1












                    $begingroup$

                    If $ln x = a_x$ and $log_{10} x = b_x$ then



                    $e^{a_x} = x$ and $10^{b_x} = x$.



                    Bear in mind $10 = e^{ln 10}$ so $10^k = (e^{ln 10})^k = e^{kln 10}$.



                    So if $10^{k_x} =e^{k_xln 10} = x$ then......



                    By definition $log_{10} x = k_x$ and $ln x = k_xln 10$ and so.......



                    $frac {ln x}{log_{10} x} = frac {k_xln 10}{k_x} = ln 10$.



                    It's just a conversion constant and shouldn't surprise us.



                    This is the very basis of the rule $log_b x = frac {log_a x}{log_a b}$ (note if $x$ is a variable and $b$ is a constant that is exactly your observation).






                    share|cite|improve this answer









                    $endgroup$
















                      1












                      1








                      1





                      $begingroup$

                      If $ln x = a_x$ and $log_{10} x = b_x$ then



                      $e^{a_x} = x$ and $10^{b_x} = x$.



                      Bear in mind $10 = e^{ln 10}$ so $10^k = (e^{ln 10})^k = e^{kln 10}$.



                      So if $10^{k_x} =e^{k_xln 10} = x$ then......



                      By definition $log_{10} x = k_x$ and $ln x = k_xln 10$ and so.......



                      $frac {ln x}{log_{10} x} = frac {k_xln 10}{k_x} = ln 10$.



                      It's just a conversion constant and shouldn't surprise us.



                      This is the very basis of the rule $log_b x = frac {log_a x}{log_a b}$ (note if $x$ is a variable and $b$ is a constant that is exactly your observation).






                      share|cite|improve this answer









                      $endgroup$



                      If $ln x = a_x$ and $log_{10} x = b_x$ then



                      $e^{a_x} = x$ and $10^{b_x} = x$.



                      Bear in mind $10 = e^{ln 10}$ so $10^k = (e^{ln 10})^k = e^{kln 10}$.



                      So if $10^{k_x} =e^{k_xln 10} = x$ then......



                      By definition $log_{10} x = k_x$ and $ln x = k_xln 10$ and so.......



                      $frac {ln x}{log_{10} x} = frac {k_xln 10}{k_x} = ln 10$.



                      It's just a conversion constant and shouldn't surprise us.



                      This is the very basis of the rule $log_b x = frac {log_a x}{log_a b}$ (note if $x$ is a variable and $b$ is a constant that is exactly your observation).







                      share|cite|improve this answer












                      share|cite|improve this answer



                      share|cite|improve this answer










                      answered 2 hours ago









                      fleabloodfleablood

                      69.4k22685




                      69.4k22685























                          0












                          $begingroup$

                          Hint: $ln(10)approx 2.3025$. Given that information, your conjecture is that $ln(x)=ln(10)log_{10}(x)$. Can you see a way to prove that?






                          share|cite|improve this answer









                          $endgroup$


















                            0












                            $begingroup$

                            Hint: $ln(10)approx 2.3025$. Given that information, your conjecture is that $ln(x)=ln(10)log_{10}(x)$. Can you see a way to prove that?






                            share|cite|improve this answer









                            $endgroup$
















                              0












                              0








                              0





                              $begingroup$

                              Hint: $ln(10)approx 2.3025$. Given that information, your conjecture is that $ln(x)=ln(10)log_{10}(x)$. Can you see a way to prove that?






                              share|cite|improve this answer









                              $endgroup$



                              Hint: $ln(10)approx 2.3025$. Given that information, your conjecture is that $ln(x)=ln(10)log_{10}(x)$. Can you see a way to prove that?







                              share|cite|improve this answer












                              share|cite|improve this answer



                              share|cite|improve this answer










                              answered 2 hours ago









                              Chris CulterChris Culter

                              20.4k43584




                              20.4k43584























                                  0












                                  $begingroup$

                                  $log_a b$ = $log_c b over log_c a$ is a general rule.
                                  Thus $log_c a = {log_c b over log_a b}$ and your expression is the case of this when $a=10, b=x, c=e$.






                                  share|cite|improve this answer











                                  $endgroup$













                                  • $begingroup$
                                    Ummm.... by your correct rule we have $$ln(10)=frac{log_x(10)}{log_x(e)}$$ which isn't what we're trying to prove.
                                    $endgroup$
                                    – Rhys Hughes
                                    2 hours ago












                                  • $begingroup$
                                    Thanks. I edited to correct
                                    $endgroup$
                                    – J. W. Tanner
                                    2 hours ago
















                                  0












                                  $begingroup$

                                  $log_a b$ = $log_c b over log_c a$ is a general rule.
                                  Thus $log_c a = {log_c b over log_a b}$ and your expression is the case of this when $a=10, b=x, c=e$.






                                  share|cite|improve this answer











                                  $endgroup$













                                  • $begingroup$
                                    Ummm.... by your correct rule we have $$ln(10)=frac{log_x(10)}{log_x(e)}$$ which isn't what we're trying to prove.
                                    $endgroup$
                                    – Rhys Hughes
                                    2 hours ago












                                  • $begingroup$
                                    Thanks. I edited to correct
                                    $endgroup$
                                    – J. W. Tanner
                                    2 hours ago














                                  0












                                  0








                                  0





                                  $begingroup$

                                  $log_a b$ = $log_c b over log_c a$ is a general rule.
                                  Thus $log_c a = {log_c b over log_a b}$ and your expression is the case of this when $a=10, b=x, c=e$.






                                  share|cite|improve this answer











                                  $endgroup$



                                  $log_a b$ = $log_c b over log_c a$ is a general rule.
                                  Thus $log_c a = {log_c b over log_a b}$ and your expression is the case of this when $a=10, b=x, c=e$.







                                  share|cite|improve this answer














                                  share|cite|improve this answer



                                  share|cite|improve this answer








                                  edited 2 hours ago

























                                  answered 2 hours ago









                                  J. W. TannerJ. W. Tanner

                                  56611




                                  56611












                                  • $begingroup$
                                    Ummm.... by your correct rule we have $$ln(10)=frac{log_x(10)}{log_x(e)}$$ which isn't what we're trying to prove.
                                    $endgroup$
                                    – Rhys Hughes
                                    2 hours ago












                                  • $begingroup$
                                    Thanks. I edited to correct
                                    $endgroup$
                                    – J. W. Tanner
                                    2 hours ago


















                                  • $begingroup$
                                    Ummm.... by your correct rule we have $$ln(10)=frac{log_x(10)}{log_x(e)}$$ which isn't what we're trying to prove.
                                    $endgroup$
                                    – Rhys Hughes
                                    2 hours ago












                                  • $begingroup$
                                    Thanks. I edited to correct
                                    $endgroup$
                                    – J. W. Tanner
                                    2 hours ago
















                                  $begingroup$
                                  Ummm.... by your correct rule we have $$ln(10)=frac{log_x(10)}{log_x(e)}$$ which isn't what we're trying to prove.
                                  $endgroup$
                                  – Rhys Hughes
                                  2 hours ago






                                  $begingroup$
                                  Ummm.... by your correct rule we have $$ln(10)=frac{log_x(10)}{log_x(e)}$$ which isn't what we're trying to prove.
                                  $endgroup$
                                  – Rhys Hughes
                                  2 hours ago














                                  $begingroup$
                                  Thanks. I edited to correct
                                  $endgroup$
                                  – J. W. Tanner
                                  2 hours ago




                                  $begingroup$
                                  Thanks. I edited to correct
                                  $endgroup$
                                  – J. W. Tanner
                                  2 hours ago











                                  0












                                  $begingroup$

                                  Some of the answers already provided get close to a full explanation, but not quite.



                                  Recall that if $b > 1$ and $x > 0$, and $$log_b x = y,$$ what this means is that $b^y = x$. In other words, the base-$b$ logarithm of $x$ is an exponent $y$ such that when the base $b$ is raised to the $y^{rm th}$ power, the result is $x$. This is the definition of a (real-valued) logarithm.



                                  So, why is it that for two bases $a$, $b$, $$frac{log_a x}{log_b x}$$ is a constant not dependent on $x$? The reason is that $log_a x$ is an exponent, say $y$, such that $a^y = x$; and $log_b x$ is an exponent, say $w$, such that $b^w = x$; then $$b^w = x = a^y.$$ And now, raising both sides to the $1/w$ power, we get $$b = (b^w)^{1/w} = (a^y)^{1/w} = a^{y/w}.$$ So the ratio $y/w$ does not depend on $x$. In fact, again using the definition of logarithm, $y/w$ is the exponent for which the base $a$ must be raised to yield $b$; that is, we explicitly have $$frac{y}{w} = log_a b,$$ and from this, we get (with one additional algebraic step) what is known as the "change-of-base" formula $$frac{log_a x}{log_a b} = log_b x.$$



                                  Note that only the definition of $log$ was used, and the rule for exponents $(b^m)^n = b^{mn}$.






                                  share|cite|improve this answer









                                  $endgroup$


















                                    0












                                    $begingroup$

                                    Some of the answers already provided get close to a full explanation, but not quite.



                                    Recall that if $b > 1$ and $x > 0$, and $$log_b x = y,$$ what this means is that $b^y = x$. In other words, the base-$b$ logarithm of $x$ is an exponent $y$ such that when the base $b$ is raised to the $y^{rm th}$ power, the result is $x$. This is the definition of a (real-valued) logarithm.



                                    So, why is it that for two bases $a$, $b$, $$frac{log_a x}{log_b x}$$ is a constant not dependent on $x$? The reason is that $log_a x$ is an exponent, say $y$, such that $a^y = x$; and $log_b x$ is an exponent, say $w$, such that $b^w = x$; then $$b^w = x = a^y.$$ And now, raising both sides to the $1/w$ power, we get $$b = (b^w)^{1/w} = (a^y)^{1/w} = a^{y/w}.$$ So the ratio $y/w$ does not depend on $x$. In fact, again using the definition of logarithm, $y/w$ is the exponent for which the base $a$ must be raised to yield $b$; that is, we explicitly have $$frac{y}{w} = log_a b,$$ and from this, we get (with one additional algebraic step) what is known as the "change-of-base" formula $$frac{log_a x}{log_a b} = log_b x.$$



                                    Note that only the definition of $log$ was used, and the rule for exponents $(b^m)^n = b^{mn}$.






                                    share|cite|improve this answer









                                    $endgroup$
















                                      0












                                      0








                                      0





                                      $begingroup$

                                      Some of the answers already provided get close to a full explanation, but not quite.



                                      Recall that if $b > 1$ and $x > 0$, and $$log_b x = y,$$ what this means is that $b^y = x$. In other words, the base-$b$ logarithm of $x$ is an exponent $y$ such that when the base $b$ is raised to the $y^{rm th}$ power, the result is $x$. This is the definition of a (real-valued) logarithm.



                                      So, why is it that for two bases $a$, $b$, $$frac{log_a x}{log_b x}$$ is a constant not dependent on $x$? The reason is that $log_a x$ is an exponent, say $y$, such that $a^y = x$; and $log_b x$ is an exponent, say $w$, such that $b^w = x$; then $$b^w = x = a^y.$$ And now, raising both sides to the $1/w$ power, we get $$b = (b^w)^{1/w} = (a^y)^{1/w} = a^{y/w}.$$ So the ratio $y/w$ does not depend on $x$. In fact, again using the definition of logarithm, $y/w$ is the exponent for which the base $a$ must be raised to yield $b$; that is, we explicitly have $$frac{y}{w} = log_a b,$$ and from this, we get (with one additional algebraic step) what is known as the "change-of-base" formula $$frac{log_a x}{log_a b} = log_b x.$$



                                      Note that only the definition of $log$ was used, and the rule for exponents $(b^m)^n = b^{mn}$.






                                      share|cite|improve this answer









                                      $endgroup$



                                      Some of the answers already provided get close to a full explanation, but not quite.



                                      Recall that if $b > 1$ and $x > 0$, and $$log_b x = y,$$ what this means is that $b^y = x$. In other words, the base-$b$ logarithm of $x$ is an exponent $y$ such that when the base $b$ is raised to the $y^{rm th}$ power, the result is $x$. This is the definition of a (real-valued) logarithm.



                                      So, why is it that for two bases $a$, $b$, $$frac{log_a x}{log_b x}$$ is a constant not dependent on $x$? The reason is that $log_a x$ is an exponent, say $y$, such that $a^y = x$; and $log_b x$ is an exponent, say $w$, such that $b^w = x$; then $$b^w = x = a^y.$$ And now, raising both sides to the $1/w$ power, we get $$b = (b^w)^{1/w} = (a^y)^{1/w} = a^{y/w}.$$ So the ratio $y/w$ does not depend on $x$. In fact, again using the definition of logarithm, $y/w$ is the exponent for which the base $a$ must be raised to yield $b$; that is, we explicitly have $$frac{y}{w} = log_a b,$$ and from this, we get (with one additional algebraic step) what is known as the "change-of-base" formula $$frac{log_a x}{log_a b} = log_b x.$$



                                      Note that only the definition of $log$ was used, and the rule for exponents $(b^m)^n = b^{mn}$.







                                      share|cite|improve this answer












                                      share|cite|improve this answer



                                      share|cite|improve this answer










                                      answered 1 hour ago









                                      heropupheropup

                                      63k66199




                                      63k66199






























                                          draft saved

                                          draft discarded




















































                                          Thanks for contributing an answer to Mathematics Stack Exchange!


                                          • Please be sure to answer the question. Provide details and share your research!

                                          But avoid



                                          • Asking for help, clarification, or responding to other answers.

                                          • Making statements based on opinion; back them up with references or personal experience.


                                          Use MathJax to format equations. MathJax reference.


                                          To learn more, see our tips on writing great answers.




                                          draft saved


                                          draft discarded














                                          StackExchange.ready(
                                          function () {
                                          StackExchange.openid.initPostLogin('.new-post-login', 'https%3a%2f%2fmath.stackexchange.com%2fquestions%2f3089210%2fwhy-is-the-ratio-of-lnx-and-logx-a-constant%23new-answer', 'question_page');
                                          }
                                          );

                                          Post as a guest















                                          Required, but never shown





















































                                          Required, but never shown














                                          Required, but never shown












                                          Required, but never shown







                                          Required, but never shown

































                                          Required, but never shown














                                          Required, but never shown












                                          Required, but never shown







                                          Required, but never shown







                                          Popular posts from this blog

                                          SQL Server 17 - Attemping to backup to remote NAS but Access is denied

                                          Always On Availability groups resolving state after failover - Remote harden of transaction...

                                          Restoring from pg_dump with foreign key constraints